Please Help Will Give Points

Answers

Answer 1

Answer:

2

Step-by-step explanation:

Answer 2
Brainleist me and ill give the answer

Related Questions

Functions, f and g are given by f(x)= 3+ cos x and g(x) = 2x, x is a real number. Determine the value of c for which f(g(x))= g(f(x)) where 0 [tex]\leq[/tex]x<2[tex]\pi[/tex]

Answers

Answer:

2 is my answer

Step-by-step explanation:

pls help me with this friends

Answers

Answer:

A,B,C

Step-by-step explanation:

if you see that two small make a square then you know three make one and a half.

A,b,c
This is the correct answer

I need help with this please!!

Answers

Answer:

8(3j -2)

4(6j -4)

Step-by-step explanation:

24j - 16

Rewriting

8*3*j - 8*2

Factor out the greatest common factor

8(3j -2)

We can also factor out 4

4*6j - 4*4

4(6j -4)

Answer:

A & C

Step-by-step explanation:

A) 8(3j-2)

= 24j-16

B) 6(4j-2j)

= 24j -12 j (not equal)

C) 4 (6j-4)

24j -16

D) 2(12-8)

24-16 (not equal)

Therefore only A and C are equivalent to 24j-16

Answered by Gauthmath

Which inequality describes this graph?

Answers

Your picture is black

umm it’s just a black page lol

William has a 26-liter glass tank. First, he wants to put some marbles in it, all of the same volume. Then, he wants to fill the tank with water until it's completely full. If he uses 85 marbles, he will have to add 20.9 liters of of water.

What is the volume of each marble?
How much water is necessary if William uses 200 marbles?

Answer: 0.06/marble | 14 liters

Answers

The volume of the marbles is   26-20.9 = 5.1 l      and there ar e85 marbles

5.1/85 = .06 l/marble

 

then 200 marbles would requrie    .06 l/marble x 200 marbles = 12 liters

so he would only need  26-12= 14 liters of water to fill the tank !

Answer:

14 marbles

Step-by-step explanation:

Hey! Can anybody help? I’m stuck on this question

Answers

I believe that it is C
it’s (C)………………………………

I need help! ASAP

( 6th grade tape charts! )

Answers

There is nothing attached
what do you need help with

what is the equation

Answers

Answer:   -12 - 3 = -15

Explanation:

We start at -12 which goes in the first box.

Moving to the left 3 units means we subtract off 3, so that's why 3 goes in the second box.

The arrow is pointing at -15 as the final destination, which goes in the final third box.

The equation is -12 -3 = -15

After school, Carlos helps his mom at her cake shop, Love and Lace Cakes. Today, he is helping her bake a 3-layer wedding cake. Each layer will be 2 1/2 inches tall and shaped like a rectangular prism. The bottom layer will be 12 inches long and 10 inches wide. The middle layer will be 10 inches long and 8 inches wide. The top layer will be 8 inches long and 6 inches wide. What will the total volume of the wedding cake be?

PLZ HURRY WILL MARK BRAINLIEST

Answers

Multiply length time width times height to find volume of each layer then add the volumes together
Height/length/Width
1st layer: 2 1/2 x 12 x 10
2nd layer: 2 1/2 x 10 x 8
3rd layer: 2 1/2 x 8 x 6
And to make the math easier, turn 2 1/2 into 2.5. They still equal the same thing, but this makes it easier to multiply. Hope this helps!

hello!! So I’m in 5th grade (middle school in my state :]) and I have a question.

what is 5 x 23 x 2 ? Please help! I didn’t understand anything in the mah lesson today and I’m behind.

if you can help that’d be a help!

also please tell me how the answer was gotten!


5 x 23 x 2

Answers

Answer:230

Step-by-step explanation:

5 x 23= 115

115 x 2= 230

Ok so this is easy let me help u
5 x 23 x 2
To make it easier let’s pick two numbers to begin with and the sum of those two numbers will be multiplied by 23
5 x 2 is the easiest to multiply so 5 x 2 = 10
And then 10 x 23 = 230
This is it I hope you understood and this helped you

what comparison does the ratio 6 to 21 represent​

Answers

Answer:

3:7

Step-by-step explanation:

hope it help you....

Answer:

2:7

Step-by-step explanation:

You have to find the common denominator and divide both numbers by that for example: 6 and 21 they can both be equally divided by 3 without becoming a fraction or a decimal.(Pls do correct me if i'm wrong don't be scared to)

25. Ava found that 5 cars passed her house in 10 minutes. Which of the following is a unit rate per hour for this situation? A 10 cars per hour 20 cars per hour B) 30 cars per hour D 40 cars per hour​

Answers

Answer:

B. 30 cars per hour

If 5 cars pass her house in 10 minutes then 30 cars will pass her house per hour

hope you got it xx

Answer :
option B 30 cars per hour

Step by step explanation:

Cars passed in 10 minutes = 5.
Cars passed in 60 minutes (one hour) = unknown

Notice how 10/2 = 5?
Same way 60/2 = 30

Hence, cars passed in 1 hour is 30 cars

Can someone help me with this two because I am very confused

Answers

Answer:

Let the height of the larger box = h

and length and width of the larger box = l and w , repectively

Now , Let find the volume of the larger box

Volume = length * width * height

So Volume of the larger box = l*w*h = lwh

Now the question says

the height of the smaller box is 80% of the height of the larger box

It means

Height of the smaller box = 80% of h = 0.80*h = 0.80h

and the other two dimensions are same so it means length and width of the smaller box = l and w

so, Volume of the smaller box = l*w*0.80h = 0.80lwh

So the Difference in the volume of the two boxes = Volume of larger box - Volume of smaller box

therefore , Difference in the Volume of two boxes = lwh - 0.80 lwh = 0.20lwh

Step-by-step explanation:

lwh=-0.80 lwh=0.80lwh

Find the arc length of the partial circle.
Either enter an exact answer in terms of π or use 3.14 π and enter your answer as a decimal.

Answers

Answer:

1/2 pi

Step-by-step explanation:

This is 1/4 of a circle

Find the  circumference and multiply by 1/4

1/4 C

1/4 ( 2*pi*r)  where r is the radius

The radius is 1

1/2 pi*1

1/2 pi

Answer:

Step-by-step explanation:

A full circle has a circumference of 2*pi * r

r = 1

so Full Circle = 2 * PI

1/4 Circle = 2*pi / 4 = 1/2 pi

The length of the arc length = 1/2 pi

Just in case you prefer the more numerical answer, 1/2 * 3.14 = 1.57

On the set of axes below, draw the graph of the equation y = -3/4x + 3. Is the point (3, 2) a solution to the equation? Explain your answer based on the graph drawn

Answers

answer: no (3,2) isn’t a solution because the line on the graph doesn’t go through it and if you plug in the ordered pair in the equation it comes out to be untrue.

(3,2) isn’t a solution because it doesn't satisfy the equation also, the line on the graph doesn’t go through it .

How can we find the solution to an equation?

We do same operations on both the sides so that equality of both expressions doesn't get disturbed.

Solving equations generally means finding the values of the variables used in it for which the considered equation is true.

The graph of the equation is given as y = -3/4x + 3.

We have to find if the point (3, 2) a solution to the equation.

So, we will subtitute the point to to satify the equation;

y = -3/4x + 3.

y = -3/4(3) + 3

y = -9 / 4 + 3

y = 3/3 = 1

Thus, (3,2) isn’t a solution because it doesn't satisfy the equation also, the line on the graph doesn’t go through it .

Learn more about solutions here:

https://brainly.com/question/15582302

#SPJ2

A person standing on a moving walkway travels 75 feet in 25 seconds. What equation represents this relationship? How many feet will the person travel in 10 seconds?

Answers

I found someone with the same question as you, good luck !
the person is travel 3 feet per second. and they will travel 30 feet in 10 seconds.

What is equivalent to -3(-2x + 7)

A. 6x - 21
B. -6x -21
C. 6x + 21
D. 13x
E. 15x

Answers

Answer is A 6x - 21 as the -3 makes the -2x positive and 7 into a negative 21
The answer would be A. 6x - 21
You’d multiply-3 and -2x which gets you a positive 6x. Then you’d multiply -3 and 7 which is -21.

If x is an integer, what is the least possible value for x that satisfies the inequality?

-5x - 12 < 33

A. -10
B. -9
C. -8
D. 9
E. 10

Answers

Answer:

C. -8

Step-by-step explanation:

We have to solve the inequality by isolating x.

-5x - 12 < 33

Add 12 to both sides.

-5x < 45

Divide both sides by -5 (remember to flip the sign when dividing by a negative).

x > -9

The least possible value for x that satisfies this inequality is -8.

The answer is none other than c witch is -9

In triangle JKL, JKL is a right angle, KM and is an altitude. JL=25 and JM=5, find KM

Answers

Answer:

50

Step-by-step explanation:

The table shows the length and width of Florida at its most distant points.

Determine the prime factorization of the length . Use "^" for the exponent and "x" for the multiplication.

Measurement. Distance (mi)

Length. 500

Width. 160

An example of prime factorization is the following:

Find prime factorization of 50.

1st find the factors of 50 until you only have prime numbers.

1x50

2x25=2x5x5

So the prime factorization of 50 is: 2x5^2

Answers

Answer:

5*10^2

Step-by-step explanation:

Length = 500

500= 5 *100

100=10^2

500= 5 *10^2

5*10^2 hope it helps!!

What is the value of the independent variable at point A on the graph?

Answers

Answer:

6

Step-by-step explanation:

The independent value refers to the x values of a graph. Point A is located at (6, 3). Therefore, the value of the independent variable is 6.

The value of the independent variable at point A on the graph is 6.

What is Independent Variable?

The cause is the independent variable. Its value is unaffected by the other study variables.

In an experimental study, an independent variable is one that you change or alter to examine its effects. It is named "independent" because it is unaffected by any other study variables.

Given:

We have two quantities from the graph one is mass in grams and another is time.

as, the masses totally depends on growth of bay with time.

So, the dependent variable is Mass and independent  variable is Time.

Now, the coordinates for Point A is (6, 3).

Hence,  the value of point A is 6.

Learn more about Independent variable here:

https://brainly.com/question/29430246

#SPJ2

What is the missing number in the input output table
Input: 3, 5, ? 17
Output: 16, 22, 31, 58

Answers

Answer:

input is 3 5 9 17 so hope you get the answer thanks

3, 5 , 7 , 9
good luck !

I don't understand this math question! The pictures provided and I'd love if someone can explain it to me

Answers

Answer:

Hello,

Step-by-step explanation:

The first number is

[tex]\dfrac{8}{12}[/tex]

The second number is

[tex]\dfrac{9}{12}[/tex]

Their sum is:

[tex]\dfrac{8}{12} +\dfrac{9}{12} =\dfrac{8+9}{12}=\dfrac{17}{12}[/tex]

Answer:

67

Step-by-step explanation:

at McDonald's, you get a $1 coupon for every 3 fries you buy. What is the least number of fries you could buy in $10 in coupons?

Answers

Answer:

You must buy at least 30 fries to get 10 dollar ins coupons

Step-by-step explanation:

We can write a ratio to find the minimum

1 dollar            10 dollars

--------------- = -------------

3 fries                x fries

Using cross products

1 *x = 10 * 3

x = 30

You must buy at least 30 fries to get 10 dollar ins coupons

Answer: 10 x 3 = 30
1 coupon= 3
10 coupon= 30

solve the following equation. answer in terms of a mixed number. 7(2x+6)=1

Answers

Answer:

[tex]7(2x + 6) = 1 \\ 14x + 42 = 1 \\ 14x = - 41 \\ x = - 2 \frac{13}{14} [/tex]

7(2x+6)=1
14x+42=1
14x=1-42
14x=-41
x=-41/14
=-2 13/14

Please help


Quadrilateral ABCD is a parallelogram. Let E be a point on AB, and let F$\ be the intersection of lines DE and BC The area of triangle EBF is 1, and the area of triangle EAD is 9. Find the area of parallelogram ABCD.

Answers

Answer:

24

Step-by-step explanation:

It is 24
Hope this is correct

Find the simple interest on \$5,000 at 4% interest for 3years.

Answers


As SI =PTR/100
SI=600$
600 hope this helps :)

Rewrite using a negative exponent.

1/3^2

Answers

Answer:

1/9 or 3^-2

Step-by-step explanation:

Ignore the value of 1/3 for now and just focus on the 3^2.

Find 3^2:

3*3 = 9

Now 9 is the denominator, therefore our answer would be 1/9 once you reassemble the fraction.

Because 1/9 is our new fraction, we can use a negative exponent to create the same result. This is because the negative exponent rule tells us that a number with a negative exponent should be put to the denominator. Therefore we can then use 3^-2.

Answer:

3^-2

Step-by-step explanation:

1. Rút gọn biểu thức A(x) = (x+2)(x+3)(x+5) - (x-1)(x+4)(x+7). Từ đó tìm tất cả các giá trị x sao cho A(x) = 0.

Answers

x = -29/7

Step-by-step explanation:

A(x) = (x^2 + 5x + 6)(x + 5) - (x^2 + 3x - 4)(x + 7)

= (x^3 + 10x^2 + 31x + 30) - (x^3 + 10x^2 + 17x - 28) = 14x + 58

A(x) = 0 => 14x + 58 =0 => x = -58/14 = -29/7

Answer:

778744

Step-by-step explanation:

387t98878989890

1. 4 < 7 Multiply both sides by 3, then by 2, then by 4, then by 9

2. 11 > -2 Add 3 to both sides, then add 3, then add (-4)

3. -2 < -2 Subtract 6 from both sides, then 8, and then 2

4. -4 < 8 Divide both sides by -4, then by -2

5. Write a short explanation of the effects of the above operations. Did this affect the inequality sign? Was it still true? Why or why not?

Answers

Answer:

1. 864 < 1512

2. 12 > 0

3. question 3 is wrong -2 = -2

4. -0.5 < 1

5. In the above operations, they did not effect the inequality sign because the same operation is being done to both sides, therefore the inequality sentences all remained true.

Step-by-step explanation:

1. (3)(4) < (7)(3)

(2)12 < 21(2)

(4)24 < 42(4)

(9)96 < 168(9)

= 864 < 1512

2. 3+11 > -2+3

3+ 13 > 1 + 3

-4+16 > 4-4

= 12 > 0

3. -2 = -2 (question 3 is wrong)

4. -4/-4 < 8/-4

1/-2 > -2/-2

= -0.5 < 1

Other Questions
can someone pls tell me what is ROC in the olympics.. I mean Ik its Russia but why is it roc and what happened [tex] \frac{3x - 2}{7} - \frac{5x - 8}{4} = \frac{1}{14} [/tex] Triangle QRP is congruent to triangle YXZ. What is the perimeter of triangle YXZ? What is the perimeter of triangle YXZ? Please help 25 PointsWhat Mathematical leads to the formulas for the magnitudes of displacement, velocity, and acceleration? . Doanh nghip d tnh u t gp vn lin doanh vi s tin ban u l 500 tr, trong vng 4 nm, li sut d kin l 12%/nm. Tnh tng s li ca hot ng u t lin doanh thu c sau khi kt thc lin doanh ? Choose the underlined partword whoseunderlined part (A,B, or D) whichneeds correctingMai's father said that she wouldrecover guickly if she follows thedoctor's advice.Mai's fatherfollowsrecover quicklydoctor's advice. what is the solution to Y=-2x+4x+8Y=-4x+16 How many moles of Cl are in 5.76 mg of FeCl3? where did the writer long for the holidays Which of the following are steps in process costing? Select one: A. Visualize the physical flow of units B. Calculate the equivalent units C. Determine the per-unit costs D. Calculate the Cost of Goods Manufactured and the Ending WIP Inventory E. All of the above A group has more than $120 to spend. The inequality 5x + 20 > 120 represents the cost to rent a hall for a conference, where x represents the number of hours the group can rent the hall. Which of the following is the solution for this situation? In an assembly-line production of industrial robots, gearbox assemblies can be installed in one minute each if holes have been properly drilled in the boxes and in ten minutes if the holes must be redrilled. Twenty-four gearboxes are in stock, 6 with improperly drilled holes. Five gearboxes must be selected from the 24 that are available for installation in the next five robots. (Round your answers to four decimal places.) (a) Find the probability that all 5 gearboxes will fit properly. (b) Find the mean, variance, and standard deviation of the time it takes to install these 5 gearboxes. speed of the beats per minute Exercise A Rewrite each of the following expressions. Use thewords divided by. Then set up the division problem. Look at theexamples on the top of this page.Miles per hour 49X =degrees What do I do Which group evolved first?raysskatessharkschimaeras please answer this questions sooni will mark him or her brillient Si un resorte de constante elstica 1300 n/m se comprime 12 cm Cuanta energa almacena? Y si estira 12cm Cuanta energa almacena? Which answer shows 0.00897 written in scientific notation?From what I heard it's not D Solve the simple interest formula, I = Prt, for P. (2 points)P = IrtP equals r over i tP equals i over r t